Đến nội dung

Hình ảnh

$\frac{x^3+y^3+z^3}{3}\geq \left ( \frac{x+y+z}{3} \right )^3$


  • Please log in to reply
Chủ đề này có 5 trả lời

#1
PBC A

PBC A

    Binh nhất

  • Thành viên
  • 32 Bài viết

Chứng minh với x,y,z không âm ta có : $\frac{x^3+y^3+z^3}{3}\geq \left ( \frac{x+y+z}{3} \right )^3$



#2
lovemath99

lovemath99

    Trung sĩ

  • Thành viên
  • 151 Bài viết

Chứng minh với x,y,z không âm ta có : $\frac{x^3+y^3+z^3}{3}\geq \left ( \frac{x+y+z}{3} \right )^3$

 

Áp dụng $Holder$ thì ta có:

 

$(1^3+1^3+1^3)(1^3+1^3+1^3)(a^3+b^3+c^3) \ge (a+b+c)^3$

 

$\to \dfrac{a^3+b^3+c^3}{3} \ge \dfrac{(a+b+c)^3}{27}$

 

$\to Q.E.D$

 

Dấu "=" xảy ra khi và chỉ khi $a=b=c$


Bài viết đã được chỉnh sửa nội dung bởi lovemath99: 05-06-2013 - 10:19


#3
PBC A

PBC A

    Binh nhất

  • Thành viên
  • 32 Bài viết

Có thể c/m bđt Holder với 3 cặp số k



#4
bachhammer

bachhammer

    Thiếu úy

  • Thành viên
  • 659 Bài viết

Giả sử $a\geq b\geq c$. Khi đó áp dụng bất đẳng thức Chebyshev ta có: $(a+b+c)(a+b+c)(a+b+c)\leq 3(a^{2}+b^{2}+c^{2})(a+b+c)\leq 9(a^{3}+b^{3}+c^{3})$. Từ đó có đpcm.


:ukliam2: TOPIC SỐ HỌC - Bachhammer :ukliam2: 

Topic số học, các bài toán về số học

:namtay  :namtay  :namtay  :lol:  :lol:  :lol:  :lol:  :excl:  :excl:  :excl:  :lol:  :lol:  :lol: :icon6:  :namtay  :namtay  :namtay  


#5
bachhammer

bachhammer

    Thiếu úy

  • Thành viên
  • 659 Bài viết

Cách khác: Áp dụng bất đẳng thức Cauchy ta có: $a^{3}+2(\frac{a+b+c}{3})^{3}\geq 3a\frac{a+b+c}{3}=a(a+b+c)$. Tương tự cho các biểu thức còn lại. Cộng lại ta có đpcm. 

Tái bút: Trên đây chỉ là một trường hợp nhỏ lẻ của bất đẳng thức sau: $\forall a_{i}\in \mathbb{R}^{+};(i=\overline{1,n})$ và n là một số nguyên dương ta luôn có bất đẳng thức:

$\frac{\sum_{i=1}^{n}a_{i}^{n}}{n}\geq (\frac{\sum_{i=1}^{n}a_{i}}{n})^{n}$


Bài viết đã được chỉnh sửa nội dung bởi bachhammer: 05-06-2013 - 10:47

:ukliam2: TOPIC SỐ HỌC - Bachhammer :ukliam2: 

Topic số học, các bài toán về số học

:namtay  :namtay  :namtay  :lol:  :lol:  :lol:  :lol:  :excl:  :excl:  :excl:  :lol:  :lol:  :lol: :icon6:  :namtay  :namtay  :namtay  


#6
andymurray44

andymurray44

    Trung sĩ

  • Thành viên
  • 153 Bài viết

$(x^{3}+y^{3}+z^{3})(x+y+z)\geq (x^{2}+y^{2}+z^{2})^{2}\geq \frac{(x+y+z)^{4}}{9}\Rightarrow Q.E.D$






1 người đang xem chủ đề

0 thành viên, 1 khách, 0 thành viên ẩn danh